1
$\begingroup$

Consider $S_\infty$ as a Coxeter group with Coxeter generators the adjacent transpositions $s_i$, $i\geq 1$. We view elements of $S_\infty$ as functions $u:\mathbb{N}\to\mathbb{N}$. Recall the Lehmer code $\mathrm{code}(u)$ is the sequence defined by $$\mathrm{code}_i(u)=| \{j>i\mid u(i)>u(j)\} |$$ A permutation $u$ is said to be dominant if $\mathrm{code}_i(u)\geq\mathrm{code}_{i+1}(u)$ for all $i$.

For a permutation $u$, associate a dominant permutation $\mu(u)$ recursively, by declaring that if $u$ is dominant, set $\mu(u)=u$, and if $u$ is not dominant, let $i$ be the maximal index such that $\mathrm{code}_i(u)<\mathrm{code}_{i+1}(u)$, and define $\mu(u)=\mu(us_i)$.

I want to prove that for any $u$, if $\mu$ is a dominant permutation such that $u\leq_R \mu$ (right weak order), then $\mu(u)\leq_R\mu$. Any ideas?

$\endgroup$

1 Answer 1

1
$\begingroup$

I found a proof. An alternative characterization of dominant permutations is that they are $132$-avoiding. Also, an alternative characterization of right weak order is that $u\leq_R v$ if and only if all left inversions that occur in $u$ also occur in $v$.

Suppose $u\leq_R\mu$ for $\mu$ dominant. If $u$ is dominant, then clearly $\mu(u) \leq_R\mu$. Otherwise we use induction on $\ell(u, \mu(u)) $. Let $i$ be the maximal index such that $\mathrm{code}_i(u) <\mathrm{code}_{i+1}(u)$. Then there exists an index $j>i+1$ such that there is a $132$ pattern at $i, i+1,j$. This pattern must not occur in $\mu$, but the existing inversions must remain. Thus $u(i+1)$ must precede $u(i) $ in $\mu$. Since this is the only left inversion that is added as we pass from $u$ to $us_i$, it follows that $us_i\leq_R\mu$. Therefore by the induction hypothesis $$\mu(u) =\mu(us_i) \leq_R \mu$$ and the result follows by induction.

$\endgroup$

You must log in to answer this question.

Start asking to get answers

Find the answer to your question by asking.

Ask question

Explore related questions

See similar questions with these tags.